GERIATRICS Flashcards

1
Q

Questions 1 and 2 pertain to the following case.
An 85-year-old man presents to the primary care clinic 1
month after the death of his spouse. His medical history
is significant for hypertension, hyperlipidemia, benign
prostatic hyperplasia (BPH), and major depressive disorder.
His current medications include metoprolol XL
25 mg daily, atorvastatin 20 mg daily, tamsulosin 0.4
mg daily, diazepam 5 mg at bedtime as needed for sleep,
and escitalopram 10 mg daily. His daughter reports that
he has been more lethargic and unsteady walking during
the past 3 days. The patient reports trouble sleeping,
necessitating the use of diazepam every night this past
week. His blood pressure is 135/72 mm Hg and heart
rate is 76 beats/minute. Urinalysis is unremarkable, thyroid-
stimulating hormone (TSH) is within the reference
range, and Geriatric Depression Scale score is 6/15.
1. Which medication is most contributing to this
patient’s lethargy and confusion?
A. Diazepam.
B. Metoprolol.
C. Atorvastatin.
D. Escitalopram.

A
  1. Answer: A
    Diazepam is a long-acting benzodiazepine that can
    accumulate in older patients, resulting in excessive lethargy,
    sedation, and unsteady gait, and the patient admits
    taking it every night during the past week (Answer A is
    correct). A worsening of the patient’s depression is evident
    with the recent bereavement; however, that would
    not explain the unsteady gait (Answer D is incorrect).
    Although metoprolol can cause lethargy, the patient’s
    current BP and metoprolol’s beta-1 selective nature
    would make metoprolol the unlikely cause of this new
    change in lethargy and unsteady gait (Answer B is
    incorrect). Atorvastatin is not a common cause of lethargy
    and confusion (Answer C is incorrect).
How well did you know this?
1
Not at all
2
3
4
5
Perfectly
2
Q

Questions 1 and 2 pertain to the following case.
An 85-year-old man presents to the primary care clinic 1
month after the death of his spouse. His medical history
is significant for hypertension, hyperlipidemia, benign
prostatic hyperplasia (BPH), and major depressive disorder.
His current medications include metoprolol XL
25 mg daily, atorvastatin 20 mg daily, tamsulosin 0.4
mg daily, diazepam 5 mg at bedtime as needed for sleep,
and escitalopram 10 mg daily. His daughter reports that
he has been more lethargic and unsteady walking during
the past 3 days. The patient reports trouble sleeping,
necessitating the use of diazepam every night this past
week. His blood pressure is 135/72 mm Hg and heart
rate is 76 beats/minute. Urinalysis is unremarkable, thyroid-
stimulating hormone (TSH) is within the reference
range, and Geriatric Depression Scale score is 6/15.

  1. Which age-related change in pharmacokinetics
    most likely underlies this patient’s medication-related
    problem?
    A. Delayed oral absorption.
    B. Decreased renal excretion.
    C. Slowed metabolism in the liver.
    D. Decreased volume of distribution.
A
  1. Answer: C
    Due to slowed liver metabolism in older adults, accumulation
    of multiple active metabolites can be expected,
    leading to increased risk of sedation and risk of falls
    (Answer C is correct). In older patients, the volume
    of distribution of lipidsoluble drugs such as diazepam
    is increased, not decreased (Answer D is incorrect).
    Diazepam is converted to active metabolites temazepam
    and further to oxazepam through CYP3A4. Oral
    absorption is not significantly altered in older adults for
    chronic medications (Answer A is incorrect). Decreased
    renal excretion is likely but is not a significant contributor
    in this patient, given the drugs on his medication list
    (Answer B is incorrect).
How well did you know this?
1
Not at all
2
3
4
5
Perfectly
3
Q

Questions 3 and 4 pertain to the following case.
A 76-year-old woman was recently admitted to a longterm
care facility for rehabilitation after several falls at
home. Her medical history is significant for hypertension,
hypothyroidism, Alzheimer disease (AD), hyperlipidemia,
and osteoarthritis (OA) of the knee. She takes
metoprolol succinate 50 mg daily, levothyroxine 75 mcg
daily, atorvastatin 10 mg daily, and donepezil 10 mg
daily. Her blood pressure is 126/80 mm Hg and heart
rate is 66 beats/minute. Basic metabolic panel results
are all within reference ranges; 25-hydroxyvitamin D concentration is 20 ng/mL, TSH is 1.89 mU/L, total
cholesterol is 180 mg/dL, low-density lipoprotein cholesterol
is 140 mg/dL, high-density lipoprotein cholesterol
is 35 mg/dL, and triglycerides is 176 mg/dL.
Her Mini–Mental State Examination (MMSE) score is
16/30, and her Geriatric Depression Scale score is 2/15.
3. Which recommendation would be most appropriate
to reduce this patient’s risk of falls?
A. Initiate memantine 5 mg daily.
B. Initiate vitamin D 1000 units daily.
C. Initiate aducanumab 1 mg/kg infusion every
4 weeks.
D. Initiate calcium carbonate 500 mg twice daily

A
  1. Answer: B
    Adding vitamin D to this resident’s regimen, given
    her deficient serum concentrations, may help reduce
    falls (Answer B is correct). Insufficient information
    is provided to determine the need to add memantine
    at this time (Answer A is incorrect). Aducanumab is
    currently only approved for MCI or mild AD, based on
    the patient’s MMSE of 16/30, the patient would be considered
    to have moderate dementia and not qualify for
    aducanumab (Answer C is incorrect). Adding calcium
    carbonate might help reduce fractures but would not
    reduce fall risk (Answer D is incorrect).
How well did you know this?
1
Not at all
2
3
4
5
Perfectly
4
Q

Questions 3 and 4 pertain to the following case.
A 76-year-old woman was recently admitted to a longterm
care facility for rehabilitation after several falls at
home. Her medical history is significant for hypertension,
hypothyroidism, Alzheimer disease (AD), hyperlipidemia,
and osteoarthritis (OA) of the knee. She takes
metoprolol succinate 50 mg daily, levothyroxine 75 mcg
daily, atorvastatin 10 mg daily, and donepezil 10 mg
daily. Her blood pressure is 126/80 mm Hg and heart
rate is 66 beats/minute. Basic metabolic panel results
are all within reference ranges; 25-hydroxyvitamin D concentration is 20 ng/mL, TSH is 1.89 mU/L, total
cholesterol is 180 mg/dL, low-density lipoprotein cholesterol
is 140 mg/dL, high-density lipoprotein cholesterol
is 35 mg/dL, and triglycerides is 176 mg/dL.
Her Mini–Mental State Examination (MMSE) score is
16/30, and her Geriatric Depression Scale score is 2/15.

  1. Which would be most appropriate for the patient’s
    osteoarthritic knee pain?
    A. Ibuprofen 200 mg four times daily.
    B. Acetaminophen 650 mg three times daily.
    C. Tramadol 50 mg three times daily as needed
    for pain.
    D. Diclofenac 1% topical gel 4 g applied to knee
    four times daily.
A
  1. Answer: B
    NSAIDs are recommended first-line for patients with
    OA. For patients greater than 75 years old with OA of the knee, topical NSAIDs are preferred over systemic
    NSAIDs due to lower risk of systemic side effects
    (Answer D is correct; Answer A is incorrect). A trial
    of acetaminophen at doses less than 3 g/day is reasonable
    for frail patients with OA of the knee after a trial
    of a topical NSAID has failed (Answer B is incorrect).
    Tramadol at doses less than 200 mg/day would be a
    reasonable alternative when more conservative medications
    have failed a trial of 1–2 weeks (Answer C is
    incorrect).
How well did you know this?
1
Not at all
2
3
4
5
Perfectly
5
Q

Questions 5–7 pertain to the following case.
An 80-year-old woman presents to your clinic accompanied
by her daughter, who no longer feels comfortable
leaving her mother alone because of her mother’s
“increasing forgetfulness.” The patient’s medical history
is significant for type 2 diabetes, hypertension,
coronary artery disease, congestive heart failure, and
OA. She takes the following medications: acetaminophen
650 mg every 6 hours as needed for pain, lisinopril
20 mg daily, furosemide 20 mg daily, potassium
chloride 20 mEq daily, carvedilol 12.5 mg twice daily,
and glipizide 5 mg daily. Her MMSE score is 18/30.
Blood tests obtained last week showed a normal basic
metabolic panel, except for a fasting plasma glucose
reading of 65 mg/dL. Her hemoglobin A1C (A1C) is
5.6%. A urinalysis is unremarkable. No nutritional
deficiencies are noted. The patient’s blood pressure is
130/80 mm Hg and heart rate is 60 beats/minute. She
receives a diagnosis of AD.

  1. Which initial intervention would be most appropriate
    to help with this patient’s cognitive function?
    A. Donepezil 10 mg daily.
    B. Galantamine extended release (ER) 24 mg
    daily.
    C. Memantine 10 mg twice daily.
    D. Rivastigmine patch 4.6 mg daily
A
  1. Answer: D
    All CIs have similar efficacy. The rivastigmine transdermal
    patch is better tolerated than oral rivastigmine,
    and 4.6 mg is the appropriate initial starting dose of
    the transdermal patch (Answer D is correct). Although
    donepezil tends to be the best tolerated CI, doses of
    cholinesterase medications should be titrated slowly
    to prevent GI upset. The initial donepezil dose is 5 mg
    daily at bedtime, and for galantamine ER, the dose
    is 8 mg once daily (Answers A and B are incorrect).
    Memantine has no beneficial effect in maintaining cognitive
    function, as measured by MMSE scores (Answer
    C is incorrect).
How well did you know this?
1
Not at all
2
3
4
5
Perfectly
6
Q

Questions 5–7 pertain to the following case.
An 80-year-old woman presents to your clinic accompanied
by her daughter, who no longer feels comfortable
leaving her mother alone because of her mother’s
“increasing forgetfulness.” The patient’s medical history
is significant for type 2 diabetes, hypertension,
coronary artery disease, congestive heart failure, and
OA. She takes the following medications: acetaminophen
650 mg every 6 hours as needed for pain, lisinopril
20 mg daily, furosemide 20 mg daily, potassium
chloride 20 mEq daily, carvedilol 12.5 mg twice daily,
and glipizide 5 mg daily. Her MMSE score is 18/30.
Blood tests obtained last week showed a normal basic
metabolic panel, except for a fasting plasma glucose
reading of 65 mg/dL. Her hemoglobin A1C (A1C) is
5.6%. A urinalysis is unremarkable. No nutritional
deficiencies are noted. The patient’s blood pressure is
130/80 mm Hg and heart rate is 60 beats/minute. She
receives a diagnosis of AD.

  1. Which intervention would be most appropriate to
    prevent an adverse drug reaction?
    A. Discontinue glipizide.
    B. Discontinue lisinopril.
    C. Reduce carvedilol to 6.25 mg twice daily.
    D. Reduce potassium chloride to 10 mEq daily.
A
  1. Answer: A
    This patient’s current fasting blood glucose of 65 mg/
    dL and A1C of 5.6% should prompt the pharmacist to
    request glipizide discontinuation (Answer A is correct).
    According to the American Diabetes Association’s
    Standards of Medical Care in Diabetes guidelines, older
    adults with several chronic illnesses, cognitive impairment,
    or functional dependence should have less stringent
    glycemic goals, such as A1C less than 8.0%–8.5%.
    The goals of therapy are to prevent hypoglycemia in
    older patients at greatest risk of this adverse drug reaction.
    There is no rationale for reducing the carvedilol
    dose, and given her normal basic metabolic panel and
    blood pressure, reducing potassium chloride or discontinuing
    lisinopril is not indicated at this time (Answers
    B–D are incorrect).
How well did you know this?
1
Not at all
2
3
4
5
Perfectly
7
Q

Questions 5–7 pertain to the following case.
An 80-year-old woman presents to your clinic accompanied
by her daughter, who no longer feels comfortable
leaving her mother alone because of her mother’s
“increasing forgetfulness.” The patient’s medical history
is significant for type 2 diabetes, hypertension,
coronary artery disease, congestive heart failure, and
OA. She takes the following medications: acetaminophen
650 mg every 6 hours as needed for pain, lisinopril
20 mg daily, furosemide 20 mg daily, potassium
chloride 20 mEq daily, carvedilol 12.5 mg twice daily,
and glipizide 5 mg daily. Her MMSE score is 18/30.
Blood tests obtained last week showed a normal basic
metabolic panel, except for a fasting plasma glucose
reading of 65 mg/dL. Her hemoglobin A1C (A1C) is
5.6%. A urinalysis is unremarkable. No nutritional
deficiencies are noted. The patient’s blood pressure is
130/80 mm Hg and heart rate is 60 beats/minute. She
receives a diagnosis of AD.

  1. One year later, the patient returns to the clinic.
    She has moved in with her daughter. Lately, she
    wanders around the house continuously. She often
    changes clothes, cries out, and asks repetitive questions.
    Her current medication regimen includes
    rivastigmine 9.5 transdermal patch daily, which
    she has been taking for the past 6 months. Which
    would be most appropriate for this patient’s new
    behavioral symptoms?
    A. Initiate olanzapine 5 mg daily.
    B. Initiate risperidone 0.5 mg twice daily.
    C. Initiate pimavanserin 34 mg daily.
    D. Change acetaminophen to 650 mg every 6
    hours around-the-clock.
A
  1. Answer: D
    Identifying reversible or underlying causes, such as
    pain or constipation, should be completed first when
    a patient experiences new behavioral changes. Given this patient’s history of OA and advancing dementia,
    they may not be able to accurately communicate their
    pain. Changing the acetaminophen from as needed to
    scheduled, should be the first intervention to rule out
    an underlying cause to this new behavior (Answer D
    is correct). The off-label use of atypical antipsychotic
    medications in patients with behavioral symptoms of
    dementia should be reserved for patients who pose a
    danger to themselves or others or experience hallucinations
    or delusions that are stressful to them (Answers
    A and B are incorrect). Currently there is no evidence
    supporting the use of pimavenserin in AD, and it should
    only be reserved for patients with Parkinson disease
    psychosis (Answer C is incorrect).
How well did you know this?
1
Not at all
2
3
4
5
Perfectly
8
Q
  1. An 80-year-old woman had a total right knee
    replacement 3 days ago after conservative strategies
    for OA failed. Her medical history is significant
    for hypothyroidism, osteoporosis, OA, and
    hyperlipidemia. Her current medications include
    simvastatin 20 mg daily, risedronate 35 mg weekly,
    levothyroxine 75 mcg daily, and oxycodone/
    acetaminophen 5/325 mg 1 tablet every 4 hours as
    needed for moderate pain. She is in the hospital
    preparing for discharge. As the pharmacist is counseling
    the patient on her discharge medication, the
    patient reports a new onset of “losing her water”
    the day before and again overnight. Which intervention
    would be most appropriate for this patient?
    A. Urinalysis.
    B. Pelvic floor exercises.
    C. Mirabegron 25 mg daily.
    D. Duloxetine 20 mg daily.
A
  1. Answer: A
    Any new symptom of UI in an older adult should be
    thoroughly evaluated to determine whether there is a
    reversible cause. Infection, or the “I” in the mnemonic
    DRIP, may be the cause of the new symptoms in this
    patient. Urinalysis would be the most appropriate
    intervention for this reversible cause of incontinence
    (Answer A is correct). Mirabegron is a reasonable
    option for urge incontinence or over-active bladder
    once reversible causes have been ruled out (Answer
    C is incorrect). Duloxetine has been used off-label for
    stress incontinence (Answer D is incorrect). Pelvic floor
    muscle exercises or Kegel exercises should be firstline
    therapy for stress, urge, or mixed incontinence in
    women (Answer B is incorrect).
How well did you know this?
1
Not at all
2
3
4
5
Perfectly
9
Q

Questions 9 and 10 pertain to the following case.
A 69-year-old man is admitted to the hospital after a
motorcycle collision. He had serious injuries resulting
in a left leg above-the-knee amputation and has undergone
several surgical procedures and rehabilitation
in the past 2 weeks. His current medications include
tamsulosin 0.4 mg daily, atenolol 25 mg daily, amlodipine
10 mg daily, senna/docusate 8.6/50 mg twice daily,
oxycodone controlled release 10 mg every 12 hours,
and hydromorphone 4 mg every 3 hours as needed for
breakthrough pain (uses 1–2 daily). His blood pressure
is 155/88 mm Hg, heart rate is 84 beats/minute,
and postvoid residual (PVR) volume is 400 mL after
voiding 110 mL. His chronic medical conditions are
unremarkable except for hypertension, BPH, and gastroesophageal
reflux disease.
9. Which intervention would be most appropriate for
this patient?
A. Change tamsulosin to alfuzosin 10 mg once
daily.
B. Increase atenolol to 50 mg daily.
C. Change tamsulosin to doxazosin 1 mg daily.
D. Reduce hydromorphone to 2 mg every 3 hours
as needed for breakthrough pain.

A
  1. Answer: C
    In this patient with comorbid conditions of hypertension
    and BPH, the choice of α-blockers is based on the adverse
    effect profiles. This patient has an elevated PVR volume,
    so changing tamsulosin to doxazosin might reduce both
    blood pressure and urinary retention; merely changing to
    another selective α-blocker might not provide adequate
    relief of both conditions. (Answer C is correct; Answer A
    is incorrect). Increasing the atenolol dose would address
    only the increased blood pressure, without affecting the
    current problem of acute urinary retention (Answer B
    is incorrect). The patient is receiving moderate doses of
    controlled-release opioid, so reducing the hydromorphone
    dose for breakthrough pain is unlikely to help
    reduce the obstruction that may be worsened by the narcotics
    (Answer D is incorrect).
How well did you know this?
1
Not at all
2
3
4
5
Perfectly
10
Q

Questions 9 and 10 pertain to the following case.
A 69-year-old man is admitted to the hospital after a
motorcycle collision. He had serious injuries resulting
in a left leg above-the-knee amputation and has undergone
several surgical procedures and rehabilitation
in the past 2 weeks. His current medications include
tamsulosin 0.4 mg daily, atenolol 25 mg daily, amlodipine
10 mg daily, senna/docusate 8.6/50 mg twice daily,
oxycodone controlled release 10 mg every 12 hours,
and hydromorphone 4 mg every 3 hours as needed for
breakthrough pain (uses 1–2 daily). His blood pressure
is 155/88 mm Hg, heart rate is 84 beats/minute,
and postvoid residual (PVR) volume is 400 mL after
voiding 110 mL. His chronic medical conditions are
unremarkable except for hypertension, BPH, and gastroesophageal
reflux disease.

    1. One year later, the patient has returned for
      complaints related to his BPH. His blood pressure
      is 118/74 mm Hg, heart rate 78 beats/minute, and
      PVR volume is 220 mL after voiding 150 mL.
      Current medications include doxazosin 4 mg daily,
      atenolol 25 mg daily, and amlodipine 10 mg daily.
      What is the most appropriate intervention for this
      patient?
      A. Initiate tadalafil 5 mg daily.
      B. Initiate finasteride 5 mg daily.
      C. Initiate alfuzosin 10 mg daily.
      D. Initiate saw palmetto supplement daily 40 mg
      injected into affected joint.
A
  1. Answer: C
    Due to continued elevated PVR volume with reported
    symptoms, therapy combination is recommended for
    this patient. Doxazosin plus finasteride is the most
    extensively studied (Answer B is correct). Tamsulosin
    plus dutasteride is also an extensively studied combination
    in BPH. Tadalafil use with doxazosin and other
    nonselective alpha1 blockers is not recommended due
    to increased risk of hypotension (Answer A is incorrect).
    Alfuzosin is a selective alpha1 blocker and should
    not be used in combination with other alpha1 blockers
    (Answer C is incorrect). Currently there is not enough
    evidence to support the efficacy of saw palmetto
    (Answer D is incorrect).
How well did you know this?
1
Not at all
2
3
4
5
Perfectly
11
Q
  1. A 72-year-old woman (height 66 inches, weight
    82 kg) whose medical history is significant for
    rheumatoid arthritis (RA), type 2 diabetes, gastroesophageal
    reflux disease, and hypothyroidism
    presents to the clinic with inflammation of
    the joints of the hands and stiffness lasting 1–2
    hours in the morning. She is a smoker. Her current

medications include pantoprazole 40 mg daily,
metformin 850 mg twice daily, levothyroxine 100
mcg daily, folic acid 1 mg daily, methotrexate 12.5
mg weekly, naproxen 500 mg twice daily, calcium
600 mg twice daily, and vitamin D 1000 units twice
daily. Her laboratory tests show a negative rheumatoid
factor (RF) but positive anti–cyclic citrullinated
peptides. The physician determines that
this is a flare of moderate disease. Which would be
the most appropriate intervention for maintenance
treatment of this patient’s RA?
A. Change naproxen to prednisone 20 mg daily.
B. Change methotrexate to 25 mg intramuscularly.
C. Change methotrexate to leflunomide 20 mg
daily.
D. Add sulfasalazine 500 mg twice daily and
hydroxychloroquine 400 mg daily.

A
  1. Answer: D
    In patients with recurring RA symptoms, moderate
    disease activity, and the presence of a poor prognostic
    factor (anti–cyclic citrullinated peptides), adding sulfasalazine
    and hydroxychloroquine to methotrexate
    follows guidelines from the 2015 American College
    of Rheumatology recommendations update for the
    treatment of RA (Answer D is correct). Specifically,
    these guidelines recommend either double- or triplecombination
    DMARD therapy for patients with an inadequate
    response to methotrexate. Prednisone may be
    used as bridge therapy, but continued therapy may not be
    supported by a risk-benefit analysis (Answer A is incorrect).
    Changing methotrexate from the oral route to the
    intramuscular route would offer no significant benefit in
    this case (Answer B is incorrect). Similarly, changing
    methotrexate to monotherapy with leflunomide would
    provide no significant benefits (Answer C is incorrect).
How well did you know this?
1
Not at all
2
3
4
5
Perfectly
12
Q
  1. A 66-year-old man is initiated on allopurinol
    100 mg once daily and naproxen 500 mg twice
    daily for the treatment of an acute gout flare. On
    diagnosis, no tophi were present, serum urate was
    10.9 mg/dL, and GFR was 78 mL/minute/1.73 m2.
    His allopurinol dose is increased 2 weeks after initial
    presentation to 200 mg, and at his 1-month follow-
    up, the allopurinol dose was further increased
    to 300 mg once daily. Naproxen was discontinued
    because his symptoms had resolved; serum
    urate was 8.7 mg/dL and GFR was 84 mL/minute/
    1.73 m2. He is seen 2 weeks later for an emergency
    follow-up because he has developed a new
    rash; serum urate is 7.4 mg/dL and GFR is 72 mL/
    minute/1.73 m2. Which is best for managing his
    gout at this time?
    A. Increasing allopurinol to 400 mg daily.
    B. Reinitiating naproxen 500 mg twice daily.
    C. Changing allopurinol to febuxostat 40 mg
    daily.
    D. Changing allopurinol to pegloticase 8 mg
    intravenously every 2 weeks.
A
  1. Answer: C
    Trial of a different XAO (e.g., febuxostat) should be
    initiated in the setting of intolerance or adverse effects
    before a uricosuric agent is tried (Answer C is correct).
    The allopurinol dose should not be increased at this
    time because of the presence of a new-onset rash. This
    may be an adverse effect associated with allopurinol;
    therefore, allopurinol should be discontinued (Answer
    A is incorrect). There is no acute need to restart NSAID
    therapy at this time because this patient’s pain from
    the acute flare has resolved, and he is not currently
    experiencing an acute attack (Answer B is incorrect).
    Pegloticase is reserved for patients with a severe gout
    burden whose disease is refractory to or intolerant of
    other ULTs (Answer D is incorrect).
How well did you know this?
1
Not at all
2
3
4
5
Perfectly
13
Q

Questions 1 and 2 pertain to the following case.
An 85-year-old woman (weight 65 kg) who resides at home with her daughter has a medical history significant for
type 2 diabetes and hypertension, and 1 year ago, she had a right hip fracture after a fall. Her regularly scheduled
medications include glyburide 10 mg daily, lisinopril 10 mg daily, metformin 500 mg twice daily, aspirin 81 mg
daily, and a multivitamin daily. Her as-needed medications include melatonin 6 mg at bedtime as needed for sleep,
meclizine 25 mg ½ tablet three times daily as needed for dizziness, and docusate 100 mg twice daily. Her laboratory
results show fasting plasma glucose 90 mg/dL, sodium (Na) 138 mEq/L, potassium (K) 4.5 mEq/L, chloride
(Cl) 102 mEq/L, carbon dioxide (CO2) 25 mEq/L, blood urea nitrogen (BUN) 30 mg/dL, SCr 1.8 mg/dL, and TSH
4.0 mU/L.
1. Considering the potential for altered pharmacokinetics, which set of medications is most likely to cause problems
for the patient?
A. Aspirin and melatonin.
B. Lisinopril and meclizine.
C. Lisinopril and metformin.
D. Glyburide and metformin.

A
  1. Answer: D
    Renal elimination is usually the most significantly
    changed pharmacokinetic value in older adults. This
    patient’s advanced age and diseases will add to her loss
    of renal function. Using the Cockcroft-Gault equation,
    this patient’s estimated CrCl is 24 mL/minute/1.73 m2.
    Creatinine clearance = [(140 − 85) × 65]/[(72 × 1.8)] ×
    0.85. At this level of function, glyburide elimination
    would be prolonged, and metformin use is contraindicated
    (Answer D is correct). The aspirin is low dose,
    and melatonin is safe even at very high doses (Answer
    A is incorrect). Although lisinopril is renally eliminated,
    dosing is based on response, and meclizine has
    mostly hepatic metabolism with no dosage adjustment
    in renal insufficiency (Answer B is incorrect). Answer
    C is incorrect because lisinopril, unlike glyburide, is
    not considered potentially inappropriate in older adults.
How well did you know this?
1
Not at all
2
3
4
5
Perfectly
14
Q

Questions 1 and 2 pertain to the following case.
An 85-year-old woman (weight 65 kg) who resides at home with her daughter has a medical history significant for
type 2 diabetes and hypertension, and 1 year ago, she had a right hip fracture after a fall. Her regularly scheduled
medications include glyburide 10 mg daily, lisinopril 10 mg daily, metformin 500 mg twice daily, aspirin 81 mg
daily, and a multivitamin daily. Her as-needed medications include melatonin 6 mg at bedtime as needed for sleep,
meclizine 25 mg ½ tablet three times daily as needed for dizziness, and docusate 100 mg twice daily. Her laboratory
results show fasting plasma glucose 90 mg/dL, sodium (Na) 138 mEq/L, potassium (K) 4.5 mEq/L, chloride
(Cl) 102 mEq/L, carbon dioxide (CO2) 25 mEq/L, blood urea nitrogen (BUN) 30 mg/dL, SCr 1.8 mg/dL, and TSH
4.0 mU/L.

  1. Considering the potential for increased pharmacodynamic sensitivity, which set of medications is most likely
    to cause problems for the patient?
    A. Aspirin and melatonin.
    B. Lisinopril and meclizine.
    C. Lisinopril and metformin.
    D. Glyburide and metformin.
A
  1. Answer: B
    Common pharmacodynamic changes associated with
    aging include impaired homeostasis for electrolytes
    with angiotensin-converting enzyme inhibitors such as
    lisinopril and increased sensitivity to anticholinergic
    adverse effects from drugs such as meclizine (Answer
    B is correct). Lisinopril, metformin, and glyburide have
    primarily pharmacokinetic problems because of renal
    excretion changes when used in older adults (Answers C
    and D are incorrect). Melatonin is extremely safe without
    pharmacodynamic or pharmacokinetic issues in
    older adults, and the aspirin is low dose, so issues with
    GI bleeding are less than with higher doses (Answer A
    is incorrect).
How well did you know this?
1
Not at all
2
3
4
5
Perfectly
15
Q

Questions 3–5 pertain to the following case.
A 70-year-old woman (height 66 inches, weight 71.7 kg [158 lb]) is in the clinic for an evaluation by the clinical pharmacist
for polypharmacy. She has complaints of fatigue, light-headedness, constipation, and “too many medicines.”
Her medical history is significant for hypertension, coronary artery disease (drug-eluting stent 8 years ago), chronic
obstructive pulmonary disease, diabetes mellitus, incontinence, frequent urinary tract infections, depression, and
moderate dementia. Vital signs include blood pressure 160/82 mm Hg, heart rate 51 beats/minute, respiratory rate
16 breaths/minute, and oxygen saturation 99% on room air. Her current medications are as follows: fluticasone/salmeterol
250/50 1 puff twice daily, aspirin 81 mg daily, acetaminophen 650 mg three times daily, clopidogrel 75 mg
daily, donepezil 10 mg daily, glipizide 5 mg twice daily, lisinopril 10 mg daily, loratadine 10 mg daily, metoprolol 50
mg twice daily, paroxetine 40 mg daily, ranitidine 150 mg twice daily, simvastatin 40 mg at bedtime, and tolterodine
2 mg at bedtime. Nitrofurantoin 100 mg twice daily for 10 days was initiated 3 days ago. Laboratory values from
her physician visit 3 days before are as follows: Na 130 mg/dL, K 4.2 mEq/dL, Cl 99 mg/dL, CO2 24 mEq/dL, BUN
24 mg/dL, SCr 1.6 mg/dL, fasting glucose 67 mg/dL, A1C 6.3%, urinalysis unremarkable except for blood- small,
pH 7.5, RBC 11–25/high-power field (HPF), white blood cells 0–2/HPF, and bacteria 168/HPF.
3. Which medication list best depicts the medications with the greatest potential to harm this patient, according
to the AGS 2019 Beers Criteria?
A. Paroxetine, donepezil, tolterodine.
B. Donepezil, glipizide, simvastatin.
C. Glipizide, donepezil, nitrofurantoin.
D. Metoprolol, clopidogrel, ranitidine.

A
  1. Answer: A
    Glipizide and simvastatin are not listed in the 2019 AGS
    Beers Criteria tables (Answer B is incorrect). In addition
    to glipizide not being listed, the patient’s CrCl is
    greater than 30 mL/minute/1.73 m2, and nitrofurantoin
    is not for long-term suppression of bacteria (Answer C
    is incorrect). Histamine-2 receptor antagonists are no
    longer recommended to avoid in patients with dementia
    or cognitive impairment because of weak evidence of
    adverse cognitive effects, and neither metoprolol nor
    clopidogrel is listed (Answer D is incorrect). However,
    paroxetine should be used with caution in patients with
    hyponatremia; donepezil as a CI should be avoided in
    patients with syncope because it can cause bradycardia; and tolterodine has strong anticholinergic properties
    and should be avoided in patients with dementia
    (Answer A is correct).
How well did you know this?
1
Not at all
2
3
4
5
Perfectly
16
Q

Questions 3–5 pertain to the following case.
A 70-year-old woman (height 66 inches, weight 71.7 kg [158 lb]) is in the clinic for an evaluation by the clinical pharmacist
for polypharmacy. She has complaints of fatigue, light-headedness, constipation, and “too many medicines.”
Her medical history is significant for hypertension, coronary artery disease (drug-eluting stent 8 years ago), chronic
obstructive pulmonary disease, diabetes mellitus, incontinence, frequent urinary tract infections, depression, and
moderate dementia. Vital signs include blood pressure 160/82 mm Hg, heart rate 51 beats/minute, respiratory rate
16 breaths/minute, and oxygen saturation 99% on room air. Her current medications are as follows: fluticasone/salmeterol
250/50 1 puff twice daily, aspirin 81 mg daily, acetaminophen 650 mg three times daily, clopidogrel 75 mg
daily, donepezil 10 mg daily, glipizide 5 mg twice daily, lisinopril 10 mg daily, loratadine 10 mg daily, metoprolol 50
mg twice daily, paroxetine 40 mg daily, ranitidine 150 mg twice daily, simvastatin 40 mg at bedtime, and tolterodine
2 mg at bedtime. Nitrofurantoin 100 mg twice daily for 10 days was initiated 3 days ago. Laboratory values from
her physician visit 3 days before are as follows: Na 130 mg/dL, K 4.2 mEq/dL, Cl 99 mg/dL, CO2 24 mEq/dL, BUN
24 mg/dL, SCr 1.6 mg/dL, fasting glucose 67 mg/dL, A1C 6.3%, urinalysis unremarkable except for blood- small,
pH 7.5, RBC 11–25/high-power field (HPF), white blood cells 0–2/HPF, and bacteria 168/HPF.

  1. Given the available patient information, which set of medications is least appropriate for this patient, according
    to the Medication Appropriateness Index?
    A. Fluticasone/salmeterol, ranitidine, donepezil, tolterodine.
    B. Metoprolol, clopidogrel, ranitidine.
    C. Aspirin, glipizide, donepezil, nitrofurantoin.
    D. Paroxetine, nitrofurantoin, simvastatin.
A
  1. Answer: B
    Fluticasone/salmeterol meets the 10 criteria for the
    Medication Appropriateness Index (Answer A is incorrect).
    Ranitidine has no listed indication in this patient,
    clopidogrel has exceeded the recommended therapy
    duration for her stent, and metoprolol has a significant
    drug-drug interaction with donepezil, given her
    bradycardia (Answer B is correct). Minus duplication
    with other drugs (clopidogrel), aspirin meets the other
    nine criteria of the Medication Appropriateness Index
    (Answer C is incorrect). Nitrofurantoin has been initiated
    for a presumptive urinary tract infection, given
    her history of frequent urinary tract infections, and
    simvastatin is appropriate for her disease and condition
    (Answer D is incorrect).
17
Q

Questions 3–5 pertain to the following case.
A 70-year-old woman (height 66 inches, weight 71.7 kg [158 lb]) is in the clinic for an evaluation by the clinical pharmacist
for polypharmacy. She has complaints of fatigue, light-headedness, constipation, and “too many medicines.”
Her medical history is significant for hypertension, coronary artery disease (drug-eluting stent 8 years ago), chronic
obstructive pulmonary disease, diabetes mellitus, incontinence, frequent urinary tract infections, depression, and
moderate dementia. Vital signs include blood pressure 160/82 mm Hg, heart rate 51 beats/minute, respiratory rate
16 breaths/minute, and oxygen saturation 99% on room air. Her current medications are as follows: fluticasone/salmeterol
250/50 1 puff twice daily, aspirin 81 mg daily, acetaminophen 650 mg three times daily, clopidogrel 75 mg
daily, donepezil 10 mg daily, glipizide 5 mg twice daily, lisinopril 10 mg daily, loratadine 10 mg daily, metoprolol 50
mg twice daily, paroxetine 40 mg daily, ranitidine 150 mg twice daily, simvastatin 40 mg at bedtime, and tolterodine
2 mg at bedtime. Nitrofurantoin 100 mg twice daily for 10 days was initiated 3 days ago. Laboratory values from
her physician visit 3 days before are as follows: Na 130 mg/dL, K 4.2 mEq/dL, Cl 99 mg/dL, CO2 24 mEq/dL, BUN
24 mg/dL, SCr 1.6 mg/dL, fasting glucose 67 mg/dL, A1C 6.3%, urinalysis unremarkable except for blood- small,
pH 7.5, RBC 11–25/high-power field (HPF), white blood cells 0–2/HPF, and bacteria 168/HPF.

  1. Which medications would best be discontinued, according to the Choosing Wisely criteria?
    A. Paroxetine, ranitidine, donepezil, tolterodine.
    B. Metoprolol, clopidogrel, ranitidine.
    C. Glipizide, donepezil, nitrofurantoin.
    D. Ranitidine, nitrofurantoin, glipizide, tolterodine.
A
  1. Answer: C
    Because the patient’s A1C is less than 7.5%, glipizide
    should be reevaluated; donepezil use in dementia
    requires periodic reassessment of risk-benefit; and
    asymptomatic bacteriuria should not be treated with
    antimicrobials; these medications should be evaluated
    for continued need and possible discontinuation,
    according to the Choosing Wisely criteria (Answer C
    is correct). Paroxetine, ranitidine, and tolterodine are
    not addressed by these criteria (Answer A is incorrect).
    Similarly, metoprolol, clopidogrel, and tolterodine are
    not addressed by these criteria (Answers B and D are
    incorrect).
18
Q

Questions 6 and 7 pertain to the following case.
A 70-year-old woman is admitted to the hospital with a broken arm after a fall. While in the hospital, she is on
bedrest most of the time, loses 2 kg (current weight 63 kg), and has trouble sleeping. She is to be discharged to
a rehabilitation facility for 2–3 weeks of therapy. Her medications at discharge are glipizide 5 mg daily, lisinopril
10 mg daily, aspirin 81 mg daily, a multivitamin daily, mirtazapine 15 mg at bedtime, calcium 500 mg twice daily,
and tramadol 25 mg every 8 hours as needed for pain.
6. When recommending medication changes for this patient, which functional assessment is most important to
evaluate?
A. IADLs.
B. Depression.
C. Pressure sores.
D. Gait and balance.

A
  1. Answer: D
    This patient had a geriatric syndrome (a fall) and hazards
    of hospitalization (decline in organ systems and
    function) that occur with many older adult patients. At
    this time, she has several risk factors for another fall,
    including a history of falls, diseases such as diabetes
    and hypertension, dizziness, and use of several drugs.
    An assessment of gait and balance would help determine
    the severity of her risk (Answer D is correct). Although
    the IADL assessment is overall good and functional, it
    does not focus on the risks associated with increased
    falls (Answer A is incorrect). Evaluating the presence
    or severity of depression or of pressure sores would not be a functional assessment, though it would affect functional
    abilities (Answers B and C are incorrect).
19
Q

Questions 6 and 7 pertain to the following case.
A 70-year-old woman is admitted to the hospital with a broken arm after a fall. While in the hospital, she is on
bedrest most of the time, loses 2 kg (current weight 63 kg), and has trouble sleeping. She is to be discharged to
a rehabilitation facility for 2–3 weeks of therapy. Her medications at discharge are glipizide 5 mg daily, lisinopril
10 mg daily, aspirin 81 mg daily, a multivitamin daily, mirtazapine 15 mg at bedtime, calcium 500 mg twice daily,
and tramadol 25 mg every 8 hours as needed for pain.

  1. To maintain and improve function in this patient, which intervention is best to implement?
    A. Add atorvastatin 10 mg daily.
    B. Increase lisinopril to 20 mg daily.
    C. Add vitamin D 1000 units twice daily.
    D. Change tramadol to naproxen 500 mg twice daily as needed for pain.
A
  1. Answer: C
    Efforts to maintain bone and muscle strength are more
    important for this patient than is primary prevention
    of cardiovascular disease with atorvastatin or lisinopril.
    Most older adults do not consume a diet rich in
    vitamin D; moreover, most older adults have less sun
    exposure and are more likely to be deficient in vitamin
    D, which is a risk factor for falls and reduced muscle
    strength. Furthermore, naproxen is not a good alternative
    for the patient because of increased risk of GI
    bleeding and worsening renal function (Answer C is
    correct). Although simvastatin and lisinopril can prevent
    complications caused by cardiovascular disease
    after extended use, they do not improve functional abilities
    in the short term (Answers A and B are incorrect).
    Pain management is important for functional status,
    but use of opioids compared with non-opioids has not
    been associated with differences in functional status
    (Answer D is incorrect).
20
Q
  1. An 84-year-old widow lives at home alone. She can perform ADLs and most IADLs with her daughter’s
    assistance. Her current medications are hydrochlorothiazide 12.5 mg daily for hypertension, tolterodine long
    acting 4 mg daily for incontinence, escitalopram 20 mg daily for depression, acetaminophen 650 mg as needed
    for arthritis, and calcium/vitamin D for prevention of osteoporosis. The patient’s physician administers the
    MMSE, and her score is 23/30. On physical examination, no cogwheel rigidity or tremor is noted. Which recommendation
    would be best at this time?
    A. Add donepezil 5 mg daily.
    B. Discontinue tolterodine and reassess the patient.
    C. Add vitamin B12 1000-mg injection monthly.
    D. Change hydrochlorothiazide to lisinopril 5 mg daily.
A
  1. Answer: B
    This patient has a positive screen for mild dementia.
    However, when evaluating her cognitive loss, it is
    important to limit the use of any drug that could contribute
    to confusion, such as those identified on the AGS
    Beers Criteria, before treating for an unconfirmed condition
    (Answer A is incorrect). Anticholinergics such
    as tolterodine can cause confusion, so it would be best
    to discontinue this agent and reassess cognition before
    treating for AD (Answer B is correct). In addition, before
    initiating vitamin B12 injections, the patient should have
    laboratory evidence of deficiency (Answer C is incorrect).
    Without a serum sodium concentration, there is no
    reason to expect that hydrochlorothiazide would cause
    her cognitive decline, so changing to lisinopril is not
    indicated at this time (Answer D is incorrect).
21
Q
  1. An 87-year-old man with AD receives rivastigmine 6 mg twice daily. His family notes improved functional
    ability but reports that he has nausea and vomiting that appear to be related to rivastigmine. Which recommendation
    is best for the patient at this time?
    A. Advise the patient to take rivastigmine with an antacid.
    B. Change rivastigmine to the patch that delivers 9.5 mg daily.
    C. Discontinue rivastigmine and initiate memantine 5 mg twice daily.
    D. Add prochlorperazine 25 mg by rectal suppository with each rivastigmine dose.
A
  1. Answer: B
    Rivastigmine is a potent inhibitor of acetyl and butyryl
    cholinesterase, leading to significant cholinergic
    adverse effects such as nausea, vomiting, and diarrhea.
    However, use of the transdermal delivery system
    generates even plasma concentrations and lessens the
    incidence of cholinergic adverse effects. Because the
    maintenance dose has been achieved with rivastigmine
    12 mg, this patient can change to the patch that delivers 9.5 mg/day (Answer B is correct). Antacids
    will not substantially alleviate the GI effects of CIs, and
    prochlorperazine is anticholinergic (Answers A and D
    are incorrect). Because rivastigmine appears to work, it
    is better to continue its use, if possible, than to change
    to memantine (Answer C is incorrect).
22
Q
  1. A 75-year-old woman with AD who lives at home with her husband has been treated with donepezil 10 mg
    daily for about 3 years. When she began therapy, her MMSE score was 21/30; her present MMSE score is
    17/30. The patient cannot perform most IADLs but can perform most ADLs with cueing. About 2 months ago,
    her donepezil dose was increased to 23 mg, but she could not tolerate it, and it was reduced back to 10 mg
    daily. Her husband asks about changing her drug treatment to help maintain her function. Which is the next
    best course of action?
    A. Retry donepezil 23 mg daily.
    B. Initiate memantine 5 mg daily.
    C. Initiate aducanumab 1 mg/kg infusion every 4 weeks.
    D. Change donepezil to rivastigmine 9.5-mg patch daily.
A
  1. Answer: B
    Over 3 years, this patient’s MMSE score has decreased
    only 4 points, which suggests a treatment response
    to donepezil. Furthermore, the patient can still live
    at home with her husband, and she has maintained
    some function in her basic ADLs. However, she has
    not responded to a higher donepezil dose, and there is
    no evidence that retrying it later is useful (Answer A
    is incorrect). Changing from one CI to another is not
    effective (Answer D is incorrect). Because she has benefited
    from donepezil use, she should not abruptly discontinue
    it. Some clinical trials with memantine show
    an additional treatment response when memantine is
    added to donepezil therapy. When the benefits, risks,
    and costs have been openly discussed and the family
    prefers to consent to therapy, a time-based trial is reasonable.
    Memantine should be initiated at 5 mg daily
    (Answer B is correct). Donepezil can be evaluated for
    tapering after memantine titration. Aducanumab is
    indicated for MCI or mild AD. A MMSE of 17/30 indicates
    moderate dementia, disqualifying this patient for
    use (Answer C is incorrect).
23
Q
  1. You are evaluating the medication profile of an 87-year-old woman who resides in a secure advanced dementia
    unit. Her medical history includes dementia (likely AD), Parkinson disease, and OA. She needs assistance with
    all ADLs, including total assistance with bathing and dressing, as well as help with feeding. She transfers with
    minimal help to a wheelchair. Her medication regimen includes donepezil 10 mg daily, memantine 10 mg twice
    daily, carbidopa/levodopa 25/100 mg four times daily, and a multivitamin supplement daily. The patient’s most
    recent MMSE score is 5/30. When reviewing the nursing notes, you see several references to the patient’s continuously
    crying out, “Help me, help me,” beginning around 5 p.m. On medical evaluation, reversible causes of
    her hypervocalization are ruled out. Which initial approach is most appropriate for this patient?
    A. Initiate ibuprofen 400 mg every 8 hours.
    B. Order haloperidol 1 mg every 6 hours as needed for agitation.
    C. Begin music therapy with songs the patient enjoyed when younger.
    D. Move the patient to a private room to minimize social contacts after 3 p.m.
A

11.11. Answer: C
Patients in the late stages of dementia (as evidenced by
an MMSE score of 5/30) with behavior issues would
benefit most from nonpharmacologic treatment such as
music therapy (Answer C is correct). Social isolation
would likely increase symptomatology, and haloperidol
is not recommended until nonpharmacologic treatments
have failed or patients have become a harm to
themselves or others. In addition, the haloperidol dose
is excessive, with risk outweighing benefit (Answers
B and D are incorrect). Although pain control may be
useful, ibuprofen is not the first drug of choice and has
more risk of harm than benefit in a frail older adult
patient (Answer A is incorrect).

24
Q

You are evaluating the medication profile of an 87-year-old woman who resides in a secure advanced dementia
unit. Her medical history includes dementia (likely AD), Parkinson disease, and OA. She needs assistance with
all ADLs, including total assistance with bathing and dressing, as well as help with feeding. She transfers with
minimal help to a wheelchair. Her medication regimen includes donepezil 10 mg daily, memantine 10 mg twice
daily, carbidopa/levodopa 25/100 mg four times daily, and a multivitamin supplement daily. The patient’s most
recent MMSE score is 5/30. When reviewing the nursing notes, you see several references to the patient’s continuously
crying out, “Help me, help me,” beginning around 5 p.m. On medical evaluation, reversible causes of
her hypervocalization are ruled out. Which initial approach is most appropriate for this patient?

  1. After 2 months, the patient’s agitation increases such that the nursing staff cannot bathe or feed her. Assuming
    nonpharmacologic approaches are ineffective, which is the best pharmacologic approach to treat her behavioral
    symptoms?
    A. Increase donepezil to 23 mg daily.
    B. Begin melatonin 6 mg at bedtime.
    C. Add quetiapine 25 mg at 4 p.m. daily.
    D. Add citalopram 10 mg daily.
A
  1. Answer: C
    Increasing the dose of a CI has not been shown to reduce
    agitation with dementia (Answer A is incorrect). The
    patient has become a harm to self (because of refusing care), so a course of quetiapine is appropriate, assuming
    other nonpharmacologic treatments have been tried
    unsuccessfully (Answer C is correct). Citalopram has
    small studies showing evidence of effectiveness in the
    literature, but its role in therapy for agitation is unclear
    (Answer D is incorrect). No sleep disturbance is noted,
    so melatonin is unlikely to help (Answer B is incorrect).
25
Q
  1. A 75-year-old woman reports urinary urgency, frequency, and loss of urine when she cannot get to the bathroom
    in time. She also wears a pad at night that she changes two or three times because of incontinence. Her
    medical history is significant for MCI (MMSE score 25/30), OA, and hypothyroidism. A urinalysis is negative
    for leukocyte esterase and nitrites. Physical examination is normal, and her PVR is normal (less than 100 mL).
    Which therapy would be best to initiate for this patient at this time?
    A. Mirabegron.
    B. Darifenacin.
    C. Pelvic floor exercises and solifenacin.
    D. Pelvic floor exercises and tolterodine immediate release.
A
  1. Answer: C
    This patient has symptoms of urge incontinence. Pelvic
    floor exercises in conjunction with drug therapy should
    be offered for initial therapy (Answer C is correct).
    Darifenacin alone is not the best treatment (Answer B
    is incorrect). Some evidence indicates that solifenacin,
    a selective muscarinic blocker, does not worsen cognition,
    and solifenacin would be preferred to tolterodine
    in this patient with MCI (Answer D is incorrect).
    Mirabegron, a newer agent with less evidence for its
    exact role in therapy, should not be offered without pelvic
    floor exercises (Answer A is incorrect).
26
Q
  1. An 85-year-old man with LUTS visits his physician, who determines his AUASI score is 15. His blood pressure
    is 118/70 mm Hg sitting. A digital rectal examination confirms the diagnosis of BPH, and the physician
    schedules a further workup including a prostate ultrasound, which shows a prostate volume of 31 g. Which
    therapy is best at this time?
    A. Terazosin.
    B. Finasteride plus saw palmetto.
    C. Tamsulosin.
    D. Finasteride plus tamsulosin.
A
  1. Answer: C
    Pharmacologic therapy targeted at reducing urethral
    sphincter pressure has proved effective in reducing
    BPH symptoms. Tamsulosin is an α-adrenergic blocker
    with more specific activity for the genitourinary system.
    Given that the patient already has low normal blood
    pressure, tamsulosin would be preferred to terazosin
    (Answer C is correct; Answer A is incorrect). Orthostatic
    hypotension can still occur with all α-adrenergic
    blockers, so patients should be monitored when therapy
    is initiated. Finasteride, a 5-α-reductase inhibitor, and
    combination therapy with 5-α-reductase inhibitors are
    recommended when there is evidence of large prostate
    (Answer D is incorrect). Saw palmetto is not recommended
    in combination with 5-α-reductase inhibitors
    because it may reduce their efficacy (Answer B is
    incorrect).
27
Q
  1. An 85-year-old man presents with pain from hip OA. He has hypertension, coronary artery disease, and BPH.
    For his OA, he has been taking acetaminophen 650 mg three times daily. He reports that acetaminophen helps
    but that the pain persists and limits his ability to walk. Which is the best next step for this patient?
    A. Change acetaminophen to celecoxib.
    B. Add hydrocodone.
    C. Change acetaminophen to ibuprofen.
    D. Add glucosamine.
A
  1. Answer: B
    The AGS recommends opioids for OA when older
    patients do not respond to initial therapy with acetaminophen
    (Answer B is correct). The NSAIDs and
    COX-2 inhibitors are seldom considered when a thorough
    assessment of the patient reveals that the risk of
    treatment (GI bleeding and worsening renal function)
    does not outweigh the potential benefit (Answers A
    and C are incorrect). Glucosamine can be added to
    this patient’s medication regimen; however, even if effective, it will not provide immediate pain relief
    (Answer D is incorrect).
28
Q
  1. A 65-year-old woman received a diagnosis of RA 1 year ago. At that time, her RF titer was 1:64; she presented
    with joint inflammation in both hands and about 45 minutes of morning stiffness. She began therapy with
    oral methotrexate and currently receives methotrexate 15 mg weekly, folic acid 2 mg daily, ibuprofen 800 mg
    three times daily, and omeprazole 20 mg daily. At today’s clinic visit, the patient reports the recurrence of her
    symptoms. Radiographic evaluation of her hand joints reveals progression of joint space narrowing and bone
    erosion. Which is the next best step for treating this patient’s RA?
    A. Administer etanercept.
    B. Change to leflunomide.
    C. Add prednisone bridge therapy.
    D. Change to hydroxychloroquine.
A
  1. Answer: A
    This woman has indicators of a poor prognosis with RA
    (positive RF, many symptoms) and has not responded
    to methotrexate therapy. Although the next treatment
    step is not entirely clear, her best choices are between
    double- or triple-combination DMARD therapy and
    a biologic agent. Leflunomide or hydroxychloroquine
    would not be recommended as monotherapy for someone
    who has not responded to methotrexate (Answers B
    and D are incorrect). Etanercept has a response in 60%–
    75% of patients whose therapy with methotrexate has
    failed (Answer A is correct). Glucocorticosteroids are
    used as adjunctive therapy for the first several months
    of treatment with a disease-modifying agent and would
    be inadequate at this time (Answer C is incorrect).
29
Q

Questions 17–19 pertain to the following case.
A 60-year-old man (weight 80 kg) presents with his third gout attack of the past year. His last attack, for which he
took colchicine with good response, was 10 days ago. He is experiencing pain in his left knee and in the third and
fourth proximal interphalangeal joints on his left hand. The pain started about 10 hours ago. The patient rates his
pain as 6/10. He has chronic obstructive pulmonary disease and dyslipidemia, and his renal function is normal. His
uric acid concentration 1 month ago was 10 mg/dL. He has no tophi. His only medications are inhaled tiotropium,
albuterol, and simvastatin.
17. Which is most appropriate for treatment of this acute gout attack?
A. Naproxen 750 mg, then 250 mg every 8 hours.
B. Colchicine 1.2 mg, then 0.6 mg in 1 hour, then 0.6 mg every 12 hours.
C. Intra-articular triamcinolone injection of all affected joints.
D. Prednisone 40 mg daily plus naproxen 750 mg, then 250 mg every 8 hours.

A
  1. Answer: A
    The NSAIDs (in anti-inflammatory or acute pain
    doses), colchicine, and corticosteroids are all appropriate
    first-line therapy for acute gout. Answer A is correct
    because the listed regimen of naproxen is appropriate
    first-line therapy for acute gout. Answer B is incorrect;
    colchicine would not be recommended for this patient
    because he took acute colchicine doses in the past 2
    weeks. Intra-articular corticosteroids are recommended
    only if only one or two large joints are affected; therefore,
    Answer C is incorrect because the patient has pain
    in several small joints. This patient is having an acute
    gout attack of moderate severity. Answer D is incorrect
    because combination therapy is recommended for initial
    therapy only if the patient has a severe attack. Oral
    prednisone alone would also be appropriate; however,
    this was not a choice.
30
Q

Questions 17–19 pertain to the following case.
A 60-year-old man (weight 80 kg) presents with his third gout attack of the past year. His last attack, for which he
took colchicine with good response, was 10 days ago. He is experiencing pain in his left knee and in the third and
fourth proximal interphalangeal joints on his left hand. The pain started about 10 hours ago. The patient rates his
pain as 6/10. He has chronic obstructive pulmonary disease and dyslipidemia, and his renal function is normal. His
uric acid concentration 1 month ago was 10 mg/dL. He has no tophi. His only medications are inhaled tiotropium,
albuterol, and simvastatin.

  1. Assuming this patient will be receiving acute anti-inflammatory therapy, which decision is most appropriate
    regarding ULT in this patient?
    A. Probenecid should be initiated, but treatment should be delayed until after the acute attack has resolved.
    B. Probenecid should be initiated and can be initiated during the acute attack.
    C. Oral allopurinol should be initiated, but treatment should be delayed until after the acute attack has
    resolved.
    D. Oral allopurinol should be initiated and can be initiated during the acute attack.
A
  1. Answer: D
    Urate-lowering therapy is indicated in this patient
    because he has had two or more attacks in the past year.
    Allopurinol and febuxostat (XOIs) are first-line ULTs.
    Answers A and B are incorrect because probenecid is
    an alternative first-line ULT only if XOIs are contraindicated
    or not tolerated. Urate-lowering therapy can be
    initiated during an acute gouty attack, according to the
    ACR guidelines, as long as anti-inflammatory prophylaxis
    is instituted, which makes Answer C incorrect.
    Answer D is correct because it incorporates appropriate
    ULT in the form of allopurinol and can be initiated during the acute attack as long as the patient is also
    receiving anti-inflammatory prophylaxis.
31
Q

Questions 17–19 pertain to the following case.
A 60-year-old man (weight 80 kg) presents with his third gout attack of the past year. His last attack, for which he
took colchicine with good response, was 10 days ago. He is experiencing pain in his left knee and in the third and
fourth proximal interphalangeal joints on his left hand. The pain started about 10 hours ago. The patient rates his
pain as 6/10. He has chronic obstructive pulmonary disease and dyslipidemia, and his renal function is normal. His
uric acid concentration 1 month ago was 10 mg/dL. He has no tophi. His only medications are inhaled tiotropium,
albuterol, and simvastatin.

  1. Which regimen for anti-inflammatory prophylaxis with ULT is most appropriate for this patient once the acute
    attack has resolved?
    A. Colchicine 0.6 mg daily.
    B. Prednisone 10 mg daily.
    C. Colchicine 0.6 mg daily plus naproxen 250 mg twice daily.
    D. Pegloticase 8 mg intravenously every 2 weeks.
A
  1. Answer: A
    Oral low-dose colchicine is the first-line option for
    anti-inflammatory prophylaxis with ULT, making
    Answer A correct. Answer B is incorrect because oral
    glucocorticoids are associated with significant risks in
    long-term therapy and should not be used for anti-inflammatory
    prophylaxis unless both colchicine and
    NSAIDs are contraindicated, not tolerated, or ineffective.
    Combination therapy is not recommended for
    anti-inflammatory prophylaxis, making Answer C
    incorrect. Answer D is incorrect because pegloticase is
    used for ULT, not anti-inflammatory prophylaxis.